LSAT and Law School Admissions Forum

Get expert LSAT preparation and law school admissions advice from PowerScore Test Preparation.

User avatar
 crispycrispr
  • Posts: 71
  • Joined: Apr 08, 2021
|
#86636
Hi, I found the question stem to be misleading. It asks for what weakens "the argument that ..." when the "..." isn't the argument of the stimulus. I'm really not sure how we know which conclusion we're supposed to focus on when there are multiple.
 Adam Tyson
PowerScore Staff
  • PowerScore Staff
  • Posts: 5153
  • Joined: Apr 14, 2011
|
#87292
This argument has one standalone premise, one intermediate conclusion, and one main conclusion, crispycrispr. It is built like this:

Premise: Teen drivers cause a disproportionate share of traffic fatalities (they are 7% of drivers but responsible for 14% of fatalities, the last sentence of the stimulus.)

Intermediate Conclusion: Teen drivers lack basic driving skills. (based on the above premise; the author is arguing that the disproportionate share of fatalities is due to a lack of skill)

Main conclusion: Restrictions should be placed on teen drivers. (based on the claim that they lack basic driving skills)

The stem wants us to focus on that intermediate conclusion rather than the main conclusion. That is an argument within the stimulus and it is fair of the test makers to ask us to focus on that, even though there is another argument that they could have asked us to weaken (that restrictions should be placed on those drivers).

You asked how we are supposed to know which conclusion to focus on, but the answer is right there in the stem: they told us which one to weaken, so that's the one we pay attention to and we don't worry about the other one!

Get the most out of your LSAT Prep Plus subscription.

Analyze and track your performance with our Testing and Analytics Package.